¿Operadores ilimitados definidos solo en el subdominio denso del espacio de Hilbert en QM?

Soy relativamente nuevo en mecánica cuántica. En un conjunto de notas que estoy usando, la siguiente es una descripción de un aspecto de algunos operadores correspondientes a observables. Las notas dicen lo siguiente:

"Los observables correspondientes a operadores ilimitados no están definidos en la totalidad de H pero solo en subdominios densos de H que no son invariantes bajo la acción de los observables. Tal no varianza hace que los valores esperados, las incertidumbres y las relaciones de conmutación no estén bien definidas en la totalidad de H ."

Hay algunas cosas que no sigo. ¿Por qué sería una propiedad de los 'operadores ilimitados' que no está definido en la totalidad de H ? Además, ¿cómo entra la invariancia en esto? ¿Y cómo influye la no invariancia en los valores esperados, las incertidumbres y las relaciones de conmutación como se establece?

Respuestas (2)

Algunos operadores autoadjuntos relevantes en QM, como los proyectores ortogonales, están realmente acotados, pero estos son muy pocos en QM. La acotación es equivalente al hecho de que el rango de valores del observable, es decir, el espectro σ ( A ) del operador asociado A , está acotado en vista de la identidad del radio espectral,

| | A | | = sorber { | λ | | λ σ ( A ) } .
Sin embargo, la mayoría de los observables alcanzan valores arbitrariamente grandes (piense en los observables de posición o momento).

A su vez, la definición de operador adjunto y el teorema del gráfico cerrado prueban que la acotación de un operador autoadjunto A : D ( A ) H es equivalente a D ( A ) = H . Esto explica por qué la mayoría de los observables en QM están representados por operadores autoadjuntos cuyo dominio (siempre denso, de lo contrario el adjunto no está definido) no coincide con todo el espacio de Hilbert.

En cuanto a la invariancia del dominio, es decir, la propiedad

A ( D ( A ) ) D ( A )
el texto está un poco mal, ya que el valor esperado < A > ψ no se ve afectado por la no invariancia del dominio. Satisface, para un estado puro definido por el vector unitario ψ ,
(0) < A > ψ = ψ | A ψ .
ves eso ψ D ( A ) es suficiente para garantizar la validez de esa identidad.

Sobre las incertidumbres Δ A ψ , el texto citado puede ser correcto ya que satisfacen

(1) Δ A ψ 2 = ψ | A 2 ψ ψ | A ψ 2
y ves que la primera termia del lado derecho necesita eso A ( A ψ ) estar bien definida, es decir A ψ D ( A ) para ψ D ( A ) . Aquí, en cambio, importa la invariancia del dominio.

Finalmente, en cuanto a las relaciones de conmutación , ya que involucran composición de operadores A B y B A , las propiedades de invariancia cruzada correspondientes deben cumplir:

A ( D ( B ) ) D ( A ) y A ( D ( A ) ) D ( B ) .

Algunos comentarios finales están en orden. Estrictamente hablando, (0) no es la definición del valor esperado de A y (1) no es la definición de incertidumbre de A , en estado puro definido por el vector unitario ψ , también si son propiedades importantes. Las definiciones verdaderas respectivamente son

(2) < A > ψ := σ ( A ) λ d ψ | PAG ( A ) ( λ ) ψ
y
(3) Δ A ψ 2 := σ ( A ) ( λ < A > ψ ) 2 d ψ | PAG ( A ) ( λ ) ψ
donde he introducido la medida espectral de A , PAG ( A ) . El lado derecho de (3) está bien definido siempre que
(4) σ ( A ) λ 2 d ψ | PAG ( A ) ( λ ) ψ < +
y esta es otra forma de escribir ψ D ( A ) . Entonces, incluso en este caso, la invariancia de D ( A ) no es necesario. Obviamente (1) es válido a partir de (3) cuando ψ D ( A 2 ) y es falso en general aunque se mantiene en una forma más débil
(1') Δ A ψ 2 = A ψ | A ψ ψ | A ψ 2 .
Similarmente < A > ψ está bien definido si ψ D ( | A | ) que es una condición más débil que ψ D ( A ) .

Gracias por tu respuesta. ¿Estaría en lo cierto al afirmar que la acotación no se refiere a la definición habitual de acotación de un operador lineal sino a lo que usted describe como si el espectro es un subconjunto acotado de los números reales?
¡No, es exactamente lo mismo!
estoy usando la identidad conocida | | A | | = sorber { | λ | | λ σ ( A ) } donde ambos lados pueden (simultáneamente) ser + y en este caso A no está acotado.
Está bien, ya veo. También lo hace si D ( A ) = H solo depende de si el espectro está acotado, ¿que sea discreto o continuo no juega un papel?
Si A es autoadjunto, D ( A ) = H es equivalente al hecho de que σ ( A ) está acotado, otras características del espectro no juegan ningún papel.
Creo que el documento que estoy leyendo podría ser un poco engañoso con respecto a esto. Establece lo siguiente en la página 4: "Cuando el espectro de un observable A es discreto y A está acotado, entonces A se define en el conjunto de H y los vectores propios de A pertenece a H . En este caso, A puede verse esencialmente como una matriz. Esto significa que, en lo que respecta al espectro discreto, no hay necesidad de ampliar H ."
Ya veo, cambia de papel si no estás contento con él :)
En resumen, dice "introducción peatonal al papel del espacio de Hilbert amañado", eso es exactamente lo que estoy buscando :) Estoy estudiando QM por mi cuenta, no soy un estudiante registrado, así que necesito moverme a un ritmo lento. . De todos modos, gracias por su ayuda, sus comentarios finales están un poco avanzados en esta etapa, pero volveré sobre ellos.
Solo pude encontrar un texto de QM que usa el espacio de Hilbert amañado: Quantum Mechanics: A Modern Development . Es "nivel de posgrado" y dice que es una introducción al tema.
De hecho, los espacios de Hilbert amañados son útiles como instrumento intuitivo, pero increíblemente complicados de usar como instrumento técnico. Nunca los usé en mis artículos técnicos y en mi libro sobre teoría espectral y QM.
@ValterMoretti La fórmula del radio espectral con la que estoy familiarizado es, dada a A , dónde A es un álgebra de Banach, tenemos s pag r ( a ) = límite norte | | a norte | | 1 norte . ¿Es esta una forma diferente del mismo resultado que el tuyo?
Para operadores acotados o para C*-álgebras, la fórmula que mencionas produce el mismo resultado que la mía.

El dominio de las funciones de Hermite es invariante bajo la acción del oscilador armónico hamiltoniano y los operadores de posición y momento.

Es cierto que para operadores no acotados, "los valores esperados, las incertidumbres y las relaciones de conmutación no están bien definidas en el conjunto de H pero esto no se debe a la falta de invariancia.